Diabetes NCLEX

¡Supera tus tareas y exámenes ahora con Quizwiz!

The nurse teaches a patient with diabetes about a healthy eating plan. Which statement made by the patient indicates that teaching was successful? "I plan to lose 25 pounds this year by following a high-protein diet." "I may have a hypoglycemic reaction if I drink alcohol on an empty stomach." "I should include more fiber in my diet than a person who does not have diabetes." "If I use an insulin pump, I will not need to limit foods with saturated fat in my diet."

"I may have a hypoglycemic reaction if I drink alcohol on an empty stomach." Eating carbohydrates when drinking alcohol reduces the risk for alcohol-induced hypoglycemia. Intensified insulin therapy, such as the use of an insulin pump, allows considerable flexibility in food selection and can be adjusted for alterations from usual eating and exercise habits. However, saturated fat intake should still be limited to less than 7% of total daily calories. Daily fiber intake of 14 g/1000 kcal is recommended for the general population and for patients with diabetes. High-protein diets are not recommended for weight loss.

3. Which of the following statements are INCORRECT about exercise management for the diabetic patient? A. "I will check my blood glucose prior to exercise. If it is less than 200 I will eat a complex carb snack prior to exercising." B. "I plan on exercising for an extended period. So I will check my blood glucose prior, during, and after exercising." C. "My blood glucose is 268 and I have ketones in my urine. Therefore, I will avoid exercising today." D. All of the options are correct statements.

.3. A.

10. A patient has a blood glucose of 400. Which of the following medications could be the cause of this? A. Glyburide B. Atenolol C. Bactrim D. Prednisone

10. D.

11. A patient is scheduled to take 5 units of Humulin R and 10 units of NPH. What is the proper way of mixing these insulins? A. These insulins cannot be mixed, therefore, should be drawn up in different syringes. B. Draw-up the Humulin R insulin first and then the NPH insulin. C. Draw-up 2.5 units of NPH, then 10 units of Humulin R, and then finish drawing up 2.5 units of NPH. D. Draw-up the NPH insulin first and then the Humulin R insulin.

11. B.

13. A patient with diabetes is experiencing a blood glucose of 275 when waking. What is a typical treatment for this phenomenon? A. None, this is a normal blood glucose reading. B. The patient may need a night time dose of an intermediate-acting insulin to counteract the morning hyperglycemia. C. A bedtime snack may prevent this phenomenon. D. This is known as the Somogyi effect and requires decreasing the bedtime dose of insulin.

13. B.

14. True or False: The Somogyi effect causes the patient to experience an increase in their blood glucose during the hours of 2-3 am.

14. False

15. Which of the following insulins has no peak but a duration of 24 hours? A. NPH B. Novolog C. Lantus D. Humulin N

15. C.

18. Which of the following insulins can be administered intravenously? A. NPH B. Lantus C. Humulin R D. Novolog

18. C

2. A patient is admitted with Diabetic Ketoacidosis. The physician orders intravenous fluids of 0.9% Normal Saline and 10 units of intravenous regular insulin IV bolus and then to start an insulin drip per protocol. The patient's labs are the following: pH 7.25, Glucose 455, potassium 2.5. Which of the following is the most appropriate nursing intervention to perform next? A. Start the IV fluids and administer the insulin bolus and drip as ordered B. Hold the insulin and notify the doctor of the potassium level of 2.5 C. Hold IV fluids and administer insulin as ordered D. Recheck the glucose level

2. B

Why is 15 g of a simple sugar administered when a client with diabetes experiences hypoglycemia? 1. Inhibits glycogenesis 2. Stimulates release of insulin 3. Increases blood glucose levels 4. Provides more storage of glucose

3. Increases blood glucose levels A simple sugar provides glucose to the blood for rapid action. It does not inhibit glycogenesis. It does not stimulate the release of insulin. It does not stimulate the storage of glucose.

The nurse is assisting in sports physicals of junior high students. Which finding would prompt the nurse to ask whether a student has a history of diabetes? 1. Clustered vesicles on the lower lip 2. Silver plaques on extensor surfaces 3. Cystic acne of the face and upper back 4. A velvety darkening of the skin around the neck

4. A velvety darkening of the skin around the neck Children with type 2 diabetes often develop acanthosis nigricans, a velvety darkening of the skin around the neck, in the axillae, and in the groin. Clustered vesicles on the lip, silver plaques on extensor surfaces, and cystic acne are not particularly associated with diabetes.

4. A patient has a blood glucose of 45 and is sweating, cold, and clammy. The patient is conscious. What is your next nursing intervention? A. Recheck the blood glucose in 5 minutes. B. Give the patient 15 grams of a complex carbohydrate. C. No intervention is needed because this is a normal blood glucose. D. Give the patient 15 grams of a simple carbohydrate.

4. D.

Which results would be expected when assessing the laboratory values of a client with type 2 diabetes? 1. Ketones in the blood but not in the urine 2. Glucose in the urine but not in the blood 3. Urine and blood positive for glucose and ketones 4. Urine negative for ketones and positive glucose in the blood

4. Urine negative for ketones and positive glucose in the blood The reason for the lack of ketonuria in type 2 diabetes is unknown. One theory is that extremely high hyperglycemia and hyperosmolarity levels block the formation of ketones, stimulating lipogenesis rather than lipolysis. Ketones in the blood but not in the urine do not occur with type 2 diabetes. Glucose in the urine but not in the blood is impossible; if glycosuria is present, there must first be a level of glucose in the blood exceeding the renal threshold of 160 to 180 mg/dL (8.9−10 mmol/L). Urine and blood positive for glucose and ketones are expected in type 1 diabetes.

5. True or False: When priming the tubing for an Insulin infusion it is best practice to waste 50cc to 100cc of insulin prior to starting the infusion because insulin absorbs into the plastic lining of the tubing.

5. True

7. A patient is scheduled to take 10 units of Humulin N at 1100. When is the patient most susceptible for hypoglycemia? A. 1900 B. 1300 C. 1130 D. 1500

7. A.

6. Which patient population is most at risk for DKA?* A. Middle-aged adults who are obese B. Older-adults with Type 2 diabetes C. Newly diagnosed diabetes D. None of the options

A. Middle-aged adults who are obese

10. You are providing care to a patient experiencing diabetic ketoacidosis. The patient is on an insulin drip and their current glucose level is 300. In addition, to the insulin drip the patient also has 5% Dextrose 0.45% NS infusing in the right antecubital vein. Which of the following patient signs/symptoms causes concern?* A. Patient has a potassium level of 2.3 B. Patient complains of thirst. C. Patient is nauseous. D. Patient's skin and mucous membranes are dry

A. Patient has a potassium level of 2.3

The nurse is assigned to care for a patient diagnosed with type 2 diabetes. In formulating a teaching plan that encourages the patient to actively participate in managing diabetes, what should be the nurse's initial intervention? Assess patient's perception of what it means to have diabetes. Ask the patient to write down current knowledge about diabetes. Set goals for the patient to actively participate in managing his diabetes. Assume responsibility for all of the patient's care to decrease stress level.

Assess patient's perception of what it means to have diabetes. For teaching to be effective, the first step is to assess the patient. Teaching can be individualized after the nurse is aware of what a diagnosis of diabetes means to the patient. After the initial assessment, current knowledge can be assessed, and goals should be set with the patient. Assuming responsibility for all of the patient's care will not facilitate the patient's health.

5. A patient has an infection and reports not checking their blood glucose or regularly taking Metformin. What condition is this patient MOST at risk for?* A. HHNS B. DKA C. Metabolic alkalosis D. Metabolic acidosis

B. DKA

9. Which of the following is not a sign or symptom of Diabetic Ketoacidosis?* A. Positive Ketones in the urine B. Polydipsia C. Oliguria D. Abdominal Pain

B. Polydipsia

8. Which of the following is NOT a medical treatment for DKA and HHNS?* A. IV regular insulin B. Isotonic fluids C. Bicarbonate D. IV potassium Solution

C. Bicarbonate

The nurse is assisting a patient with newly diagnosed type 2 diabetes to learn dietary planning as part of the initial management of diabetes. The nurse would encourage the patient to limit intake of which foods to help reduce the percent of fat in the diet? Cheese Broccoli Chicken Oranges

Cheese Cheese is a product derived from animal sources and is higher in fat and calories than vegetables, fruit, and poultry. Excess fat in the diet is limited to help avoid macrovascular changes.

3. Hyperglycemic Hyperosmolar Nonketotic Syndrome would have all of the following signs and symptoms EXCEPT?* A. Dry mucous membranes B. Polyuria C. Blood glucose >600 mg/dL D. Kussmaul breathing

D. Kussmaul breathing

The nurse is assessing a patient newly diagnosed with type 2 diabetes. Which symptom reported by the patient correlates with the diagnosis? Excessive thirst Gradual weight gain Overwhelming fatigue Recurrent blurred vision

Excessive thirst The classic symptoms of diabetes are polydipsia (excessive thirst), polyuria, (excessive urine output), and polyphagia (increased hunger). Weight gain, fatigue, and blurred vision may all occur with type 2 diabetes, but are not classic manifestations.

A patient, admitted with diabetes, has a glucose level of 580 mg/dL and a moderate level of ketones in the urine. As the nurse assesses for signs of ketoacidosis, which respiratory pattern would the nurse expect to find? Central apnea Hypoventilation Kussmaul respirations Cheyne-Stokes respirations

Kussmaul respirations In diabetic ketoacidosis, the lungs try to compensate for the acidosis by blowing off volatile acids and carbon dioxide. This leads to a pattern of Kussmaul respirations, which are deep and nonlabored. Central apnea occurs because the brain temporarily stops sending signals to the muscles that control breathing, which is unrelated to ketoacidosis. Hypoventilation and Cheyne-Stokes respirations do not occur with ketoacidosis.

A patient with diabetes who has multiple infections every year needs a mitral valve replacement. What is the most important preoperative teaching the nurse should provide to prevent a cardiac infection postoperatively? Avoid sick people and wash hands. Obtain comprehensive dental care. Maintain hemoglobin A1C below 7%. Coughing and deep breathing with splinting

Obtain comprehensive dental care. A person with diabetes is at high risk for postoperative infections. The most important preoperative teaching to prevent a postoperative infection in the heart is to have the patient obtain comprehensive dental care because the risk of septicemia and infective endocarditis increases with poor dental health. Avoiding sick people, hand washing, maintaining hemoglobin A1C below 7%, and coughing and deep breathing with splinting would be important for any type of surgery but are not the priority for this patient with mitral valve replacement.

2. Type 1 diabetics typically have the following clinical characteristics:* A. Thin, young with ketones present in the urine B. Overweight, young with no ketones present in the urine C. Thin, older adult with glycosuria D. Overweight, adult-aged with ketones present in the urine

The answer is A.

6. A 36-year-old male is newly diagnosed with Type 2 diabetes. Which of the following treatments do you expect the patient to be started on initially?* A. Diet and exercise regime B. Metformin BID by mouth C. Regular insulin subcutaneous D. None, monitoring at this time is sufficient enough

The answer is A.

7. True or False: Osmotic diuresis is present in HHNS and DKA due to the kidney's inability to reabsorb the excessive glucose which causes glucose to leak into the urine which in turn causes extra water and electrolytes to be excreted.* True False

True

2. A patient is found to have a blood glucose of 375 mg/dL, positive ketones in the urine, and blood pH of 7.25. Which condition is this?* a. Diabetic Ketoacidosis (DKA) b. Hyperglycemic Hyperosmolar Nonketotic Syndrome (HHNS)

a. Diabetic Ketoacidosis (DKA)

1. This complication is found mainly in Type 2 diabetics? a. Diabetic Ketoacidosis (DKA) b. Hyperglycemic Hyperosmolar Nonketotic Syndrome (HHNS)

b. Hyperglycemic Hyperosmolar Nonketotic Syndrome (HHNS)

4. This condition happens gradually and is more likely to affect older adults?* a. Diabetic Ketoacidosis (DKA) b. Hyperglycemic Hyperosmolar Nonketotic Syndrome (HHNS)

b. Hyperglycemic Hyperosmolar Nonketotic Syndrome (HHNS)

17. When is a patient most susceptible to hypoglycemic symptoms after the administration of insulin? A. Onset B. Peak C. Duration D. Duration & Peak

17. B.

1. Which of the following symptoms do NOT present in hyperglycemia?* A. Extreme thirst B. Hunger C. Blood glucose <60 mg/dL D. Glycosuria

The answer is C.

7. What type of insulin do you expect the doctor to order for treatment of DKA? A. IV NPH B. IV Novolog C. IV Levemir D. IV Regular Insulin

7. D

9. A patient taking the medication Precose asks when it is the best time to take this medication. Your response is: A. With the first bite of food B. 1 hour prior to eating C. 1 hour after eating D. At bedtime

9. A.

10. A Type 2 diabetic may have all the following signs or symptoms EXCEPT:* A. Blurry vision B. Ketones present in the urine C. Glycosuria D. Poor wound healing

The answer is B.

16. A patient is scheduled to take 7 units of Humulin R at 0830. You administer Humulin R at 0900 in the right thigh. When do you expect this medication to peak? A. 1300 B. 0930 C. 1100 D. 1700

16. C.

A patient with diabetes is scheduled for a fasting blood glucose level at 8:00 AM. The nurse teaches the patient to only drink water after what time? 6:00 PM on the evening before the test Midnight before the test 4:00 AM on the day of the test 7:00 AM on the day of the test

Midnight before the test Typically, a patient is ordered to be NPO for 8 hours before a fasting blood glucose level. For this reason, the patient who has a lab draw at 8:00 AM should not have any food or beverages containing any calories after midnight.

A patient is newly diagnosed with type 1 diabetes and reports a headache, changes in vision, and being anxious but does not have a portable blood glucose monitor present. Which action should the nurse advise the patient to take? Eat a piece of pizza. Drink some diet pop. Eat 15 g of simple carbohydrates. Take an extra dose of rapid-acting insulin.

Eat 15 g of simple carbohydrates .When a patient with type 1 diabetes is unsure about the meaning of the symptoms they are experiencing, they should treat for hypoglycemia to prevent seizures and coma from occurring. Have the patient check the blood glucose as soon as possible. The fat in the pizza and the diet pop would not allow the blood glucose to increase to eliminate the symptoms. The extra dose of rapid-acting insulin would further decrease the blood glucose.

The nurse is reviewing laboratory results for a patient with a 15-year history of type 2 diabetes. Which result reflects the expected pattern accompanying macrovascular disease as a complication of diabetes? Increased triglyceride levels Increased high-density lipoproteins (HDL) Decreased low-density lipoproteins (LDL) Decreased very-low-density lipoproteins (VLDL)

Increased triglyceride levels Macrovascular complications of diabetes include changes to large- and medium-sized blood vessels. They include cerebrovascular, cardiovascular, and peripheral vascular disease. Increased triglyceride levels are associated with these macrovascular changes. Increased HDL, decreased LDL, and decreased VLDL are positive in relation to atherosclerosis development.

Which treatment would the nurse suggest to an adolescent with type 1 diabetes if an insulin reaction is experienced while the adolescent is at a basketball game? 1. "Call your parents immediately." 2. "Buy a soda and hamburger to eat." 3. "Administer insulin as soon as possible." 4. "Leave the arena and rest until the symptoms subside."

2. "Buy a soda and hamburger to eat."

8. A patient diagnosed with diabetes mellitus is being discharged home and you are teaching them about preventing DKA. What statement by the patient demonstrates they understood your teaching about this condition? A. "I will hold off taking my insulin while I'm sick." B. "It is normal for my blood sugar to be 250-350 mg/dL while I'm sick." C. "It is important I check my blood glucose every 3-4 hours when I'm sick and consume liquids." D. "I should not be alarmed if ketones are present in my urine because this is expected during illness."

8. C

11. A patient is being discharged home after recovering from HHNS. Which statement by the patient requires patient re-education about this condition?* A. "I will monitor my blood glucose levels regularly." B. "This condition happens suddenly without any warning signs." C. "If I become sick I will monitor my blood glucose more frequently and drink lots of fluids." D. "It is important I take my medication as prescribed."

B. "This condition happens suddenly without any warning signs."

A patient with type 2 diabetes has a urinary tract infection (UTI), is difficult to arouse, and has a blood glucose of 642 mg/dL. When the nurse assesses the urine, there are no ketones present. What nursing action is appropriate at this time? Routine insulin therapy and exercise Administer a different antibiotic for the UTI Cardiac monitoring to detect potassium changes Administer IV fluids rapidly to correct dehydration

Cardiac monitoring to detect potassium changes This patient has manifestations of hyperosmolar hyperglycemic syndrome (HHS). Cardiac monitoring will be needed because of the changes in the potassium level related to fluid and insulin therapy and the osmotic diuresis from the elevated serum glucose level. Routine insulin would not be enough, and exercise could be dangerous for this patient. Extra insulin will be needed. The type of antibiotic will not affect HHS. There will be a large amount of IV fluid administered, but it will be given slowly because this patient is older and may have cardiac or renal compromise, requiring hemodynamic monitoring to avoid fluid overload during fluid replacement.

The nurse is teaching a patient who has diabetes about vascular complications of diabetes. What information is appropriate for the nurse to include? Macroangiopathy only occurs in patients with type 2 diabetes who have severe disease. Microangiopathy most often affects the capillary membranes of the eyes, kidneys, and skin. Macroangiopathy causes slowed gastric emptying and the sexual impotency experienced by most patients with diabetes. Renal damage resulting from changes in large- and medium-sized blood vessels can be prevented by careful glucose control.

Microangiopathy most often affects the capillary membranes of the eyes, kidneys, and skin. Microangiopathy occurs in diabetes. When it affects the eyes, it is called diabetic retinopathy. When the kidneys are affected, the patient has nephropathy. When the skin is affected, it can lead to diabetic foot ulcers. Macroangiopathy can occur in either type 1 or type 2 diabetes and contributes to cerebrovascular, cardiovascular, and peripheral vascular disease. Sexual impotency and slowed gastric emptying result from microangiopathy and neuropathy.

3. A patient with diabetes has a morning glucose of 50. The patient is sweaty, cold, and clammy. Which of the following nursing interventions is the MOST important?* A. Recheck the glucose level B. Give the patient ½ cup (4 oz) of fruit juice C. Call the doctor D. Keep the patient nothing by mouth

The answer is B .

7. Which of the following statements are true regarding Type 2 diabetes treatment?* A. Insulin and oral diabetic medications are administered routinely in the treatment of Type 2 diabetes. B. Insulin may be needed during times of surgery or illness. C. Insulin is never taken by the Type 2 diabetic. D. Oral medications are the first line of treatment for newly diagnosed Type 2 diabetics

The answer is B.

A patient is admitted with diabetes, malnutrition, cellulitis, and a potassium level of 5.6 mEq/L. The nurse understands that what could be contributing factors for this laboratory result? (Select all that apply.) The level is consistent with renal insufficiency from renal nephropathy. The level may be high because of dehydration that accompanies hyperglycemia. The level may be raised due to metabolic ketoacidosis caused by hyperglycemia. The patient may be excreting sodium and retaining potassium from malnutrition. This level shows adequate treatment of the cellulitis and acceptable glucose control.

The level is consistent with renal insufficiency from renal nephropathy. The level may be high because of dehydration that accompanies hyperglycemia. The level may be raised due to metabolic ketoacidosis caused by hyperglycemia. The additional stress of cellulitis may lead to an increase in the patient's serum glucose levels. Dehydration may cause hemoconcentration, resulting in elevated serum readings. The kidneys may have difficulty excreting potassium if renal insufficiency exists. Finally, the nurse must consider the potential for metabolic ketoacidosis because potassium will leave the cell when hydrogen enters in an attempt to compensate for a low pH. Malnutrition does not cause sodium excretion accompanied by potassium retention. Thus, it is not a contributing factor to this patient's potassium level. The increased potassium level does not show adequate treatment of cellulitis or acceptable glucose control.

1. Which of the following patient statements about the diabetic diet regime is correct? A. "I'll try to consume about 20% carbs and 40% fats on a daily basis." B. "Foods that are high in mono and poly fats are avocadoes, olives, and nuts." C. "Meats increase the glycemic index; therefore, I should only consume 5% of them on a daily basis." D. "I should completely avoid starchy vegetables like potatoes and corn."

1. B

The nurse teaches a patient recently diagnosed with type 1 diabetes about insulin administration. Which statement by the patient requires an intervention by the nurse? "I will discard any insulin bottle that is cloudy in appearance." "The best injection site for insulin administration is in my abdomen." "I can wash the site with soap and water before insulin administration." "I may keep my insulin at room temperature (75° F) for up to 1 month."

"I will discard any insulin bottle that is cloudy in appearance." Intermediate-acting insulin and combination-premixed insulin will be cloudy in appearance. Routine hygiene such as washing with soap and rinsing with water is adequate for skin preparation for the patient during self-injections. Insulin vials that the patient is currently using may be left at room temperature for up to 4 weeks unless the room temperature is higher than 86° F (30° C) or below freezing (<32°F [0°C]). Rotating sites to different anatomic sites is no longer recommended. Patients should rotate the injection within one particular site, such as the abdomen.

The patient received regular insulin 10 units subcutaneously at 8:30 PM for a blood glucose level of 253 mg/dL. The nurse plans to monitor this patient for signs of hypoglycemia at which time related to the insulin's peak action? 8:40 PM to 9:00 PM 9:00 PM to 11:30 PM 10:30 PM to 1:30 AM 12:30 AM to 8:30 AM

10:30 PM to 1:30 AM Regular insulin exerts peak action in 2 to 5 hours, making the patient most at risk for hypoglycemia between 10:30 PM and 1:30 AM. Rapid-acting insulin's onset is between 10 and 30 minutes with peak action and hypoglycemia most likely to occur between 9:00 PM and 11:30 PM. With intermediate acting insulin, hypoglycemia may occur from 12:30 AM to 8:30 AM.

The nurse is preparing a teaching plan for a patient with type 2 diabetes who has been prescribed albiglutide. Which key points would the nurse include? Select all that apply. 1. The drug works in the intestine in response to food intake and acts with insulin for glucose regulation. 2. This drug increases cellular utilization of glucose, which lowers blood glucose levels. 3. This drug is used with diet and exercise to improve glycemic control in adults with type 2 diabetes. 4. The drug is an oral insulin that should be given only when the patient has something to eat immediately available. 5. Albiglutide is administered by the subcutaneous route once a week. 6. Albiglutide should be given with caution for a patient with a history of pancreatic problems.

1,3,5 Albiglutide is an incretin mimetic. These drugs work like the natural "gut" hormones, glucagon-like peptide-1 (GLP-1) and glucose-dependent insulinotropic polypeptide (GIP), that are released by the intestine in response to food intake and act with insulin for glucose regulation. They are used in addition to diet and exercise to improve glycemic control in adults with type 2 diabetes. Albiglutide is administered subcutaneously once a week.

The nurse is discussing insulin needs with an adolescent with recently diagnosed type 1 diabetes. Which information is important for the nurse to include concerning insulin administration? 1. Insulin will be required throughout life. 2. Insulin may be taken orally until adulthood. 3. Insulin needs increase with strenuous exercise. 4. Insulin needs decrease in the presence of an infection.

1. Insulin will be required throughout life. All those involved must understand that the child does not have an endogenous source of insulin and will require insulin administration throughout life. At this time there is no oral insulin with which to treat type 1 diabetes. Insulin needs decrease, not increase, during exercise. Insulin needs increase, not decrease, in the presence of an infection.

In the care of a patient with type 2 diabetes, which actions should the nurse delegate to an unlicensed assistive personnel (UAP)? Select all that apply. 1. Providing the patient with extra packets of artificial sweetener for coffee 2. Assessing how well the patient's shoes fit 3. Recording the liquid intake from the patient's breakfast tray 4. Teaching the patient what to do if dizziness or lightheadedness occurs 5. Checking and recording the patient's blood pressure 6. Assisting the patient to ambulate to the bathroom

1. Providing the patient with extra packets of artificial sweetener for coffee 3. Recording the liquid intake from the patient's breakfast tray 5. Checking and recording the patient's blood pressure 6. Assisting the patient to ambulate to the bathroom Giving the patient extra sweetener, recording oral intake, assisting with ambulation, and checking blood pressure are all within the scope of practice of the UAP. Assessing shoe fit and patient teaching are within the professional nurse's scope of practice.

The plan of care for a patient with diabetes includes all of these interventions. Which intervention should the nurse delegate to unlicensed assistive personnel (UAP)? 1. Reminding the patient to put on well-fitting shoes before ambulating 2. Discussing community resources for diabetic outpatient care 3. Teaching the patient to perform daily foot inspection 4. Assessing the patient's technique for drawing insulin into a syringe

1. Reminding the patient to put on well-fitting shoes before ambulating Reminding the patient to put on well-fitting shoes (after the nurse has taught the patient about the importance of this action) is part of assisting with activities of daily living and is within the education and scope of practice of the UAP. It is a safety measure that can prevent injury. Discussing community resources, teaching, and assessing require a higher level of education and are appropriate to the scope of practice of licensed nurses.

8. You administered 5 units of Humalog at 0800. What is the ONSET and DURATION of this medication? A. Onset: 15 minutes, Duration: 3 hours B. Onset: 2 hours, Duration: 16 hours C. Onset: 30 minutes, Duration: 1 hour D. Onset: 2 hours, Duration: 24 hours

8. A.

The nurse has been teaching a patient with diabetes how to perform self-monitoring of blood glucose (SMBG). During evaluation of the patient's technique, the nurse identifies a need for additional teaching when the patient does what? Chooses a puncture site in the center of the finger pad. Washes hands with soap and water to cleanse the site to be used. Warms the finger before puncturing the finger to obtain a drop of blood. Tells the nurse that the result of 110 mg/dL indicates good control of diabetes.

Chooses a puncture site in the center of the finger pad. The patient should select a site on the sides of the fingertips, not on the center of the finger pad because this area contains many nerve endings and would be unnecessarily painful. Washing hands, warming the finger, and knowing the results that indicate good control all show understanding of the teaching.

8. What statement or statements are INCORRECT regarding Diabetic Ketoacidosis?* A. DKA occurs mainly in Type 1 diabetics. B. Ketones are present in the urine in DKA. C. Cheyne-stokes breathing will always present in DKA. D. Severe hypoglycemia is a hallmark sign in DKA. E. Options C & D

The answer is E.

1. Which of the following is not a sign or symptom of Diabetic Ketoacidosis? A. Positive Ketones in the urine B. Oliguria C. Polydipsia D. Abdominal Pain

1. B

The nurse caring for a patient hospitalized with diabetes would look for which laboratory test result to obtain information on the patient's past glucose control? Prealbumin level Urine ketone level Fasting glucose level Glycosylated hemoglobin level

Glycosylated hemoglobin level A glycosylated hemoglobin level detects the amount of glucose that is bound to red blood cells (RBCs). When circulating glucose levels are high, glucose attaches to the RBCs and remains there for the life of the blood cell, which is approximately 120 days. Thus, the test can give an indication of glycemic control over approximately 2 to 3 months. The prealbumin level is used to establish nutritional status and is unrelated to past glucose control. The urine ketone level will only show that hyperglycemia or starvation is probably currently occurring. The fasting glucose level only indicates current glucose control.

9. A patient who has diabetes is nothing by mouth as prep for surgery. The patient states they feel like their blood sugar is low. You check the glucose and find it to be 52. The next nursing intervention would be to:* A. Administer Dextrose 50% IV per protocol B. Continue to monitor the glucose C. Give the patient 4 oz of fruit juice D. None, this is a normal blood glucose reading

The answer is A. This question requires critical thinking because the patient is NPO for surgery and can NOT eat but is experiencing hypoglycemia. Normally, you could give the patient 15 grams of a simple carbohydrate like 4 oz of fruit juice or soda, glucose tablets, gel etc. per hypoglycemia protocol However, the patient can NOT eat due to surgery prep. Therefore the nurse would need to administer Dextrose 50% IV per protocol to help increase the blood glucose and recheck the glucose level.

A client with diabetes asks the nurse whether the new forearm stick glucose monitor gives the same results as a finger stick. Which is an appropriate response to this question? 1. "There is no difference between readings." 2. "These types of monitors are meant for children." 3. "Readings are on a different scale for each monitor." 4. "Faster readings can be obtained from a finger stick."

1. "There is no difference between readings." The forearm glucose monitor is calibrated to be consistent with results obtained from a finger stick. Individuals of all ages can use these glucose monitors. A different scale is not used for each monitor; accompanying literature will indicate whether the monitor reading reflects venous blood values even though capillary blood is used. There is no difference in the time required to complete the test.

5. The _____ ______ secrete insulin which are located in the _______.* A. Alpha cells, liver B. Alpha cells, pancreas C. Beta cells, liver D. Beta cells, pancreas

The answer is D.

The nurse is teaching a patient with type 2 diabetes how to prevent diabetic nephropathy. Which statement made by the patient indicates that teaching has been successful? "Smokeless tobacco products decrease the risk of kidney damage." "I can help control my blood pressure by avoiding foods high in salt." "I should have yearly dilated eye examinations by an ophthalmologist." "I will avoid hypoglycemia by keeping my blood sugar above 180 mg/dL."

"I can help control my blood pressure by avoiding foods high in salt." Patients with type 2 diabetes to have a dilated eye examination by an ophthalmologist or a specially trained optometrist at the time of diagnosis and annually thereafter for early detection and treatment. Diabetic nephropathy is a microvascular complication associated with damage to the small blood vessels that supply the glomeruli of the kidney. Risk factors for the development of diabetic nephropathy include hypertension, genetic predisposition, smoking, and chronic hyperglycemia. Patients with type 2 diabetes need to have a dilated eye examination by an ophthalmologist or a specially trained optometrist at the time of diagnosis and annually thereafter for early detection and treatment of retinopathy.

The nurse has taught a patient admitted with diabetes principles of foot care. The nurse evaluates that the patient understands the instructions if the patient makes what statement? "I should only walk barefoot in nice dry weather." "I should look at the condition of my feet every day." "I will need to cut back the number of times I shower per week." "My shoes should fit nice and tight because they will give me firm support."

"I should look at the condition of my feet every day." Patients with diabetes need to inspect their feet daily for broken areas that are at risk for infection and delayed wound healing. Properly fitted (not tight) shoes should be worn at all times. Routine care includes regular bathing.

The nurse is teaching a patient with type 2 diabetes about exercise to help control blood glucose. The nurse knows the patient understands when the patient elicits which exercise plan? "I will go running when my blood sugar is too high to lower it." "I will go fishing frequently and pack a healthy lunch with plenty of water." "I do not need to increase my exercise routine since I am on my feet all day at work." "I will take a brisk 30-minute walk 5 days/wk and do resistance training 3 times a week."

"I will take a brisk 30-minute walk 5 days/wk and do resistance training 3 times a week." The best exercise plan for the person with type 2 diabetes is for 30 minutes of moderate activity 5 days/wk and resistance training 3 times a week. Brisk walking is moderate activity. Fishing and walking at work are light activity, and running is considered vigorous activity.

A patient admitted with type 2 diabetes asks the nurse what "type 2" means. What is the most appropriate response by the nurse? "With type 2 diabetes, the body of the pancreas becomes inflamed." "With type 2 diabetes, the patient is totally dependent on an outside source of insulin." "With type 2 diabetes, insulin secretion is decreased, and insulin resistance is increased." "With type 2 diabetes, the body produces autoantibodies that destroy β-cells in the pancreas."

"With type 2 diabetes, insulin secretion is decreased, and insulin resistance is increased." In type 2 diabetes, the secretion of insulin by the pancreas is reduced and/or the cells of the body become resistant to insulin. The pancreas becomes inflamed with pancreatitis. The patient is totally dependent on exogenous insulin and may have had autoantibodies destroy the β-cells in the pancreas with type 1 diabetes.

A client with recently diagnosed diabetes states, "I feel bad. My spouse and I do not get along. It seems as though my spouse doesn't care about my diabetes." Which response by the nurse is appropriate? 1. "You don't get along with your spouse." 2. "I'm sorry. What can I do to make you feel better?" 3. "It may be temporary because your spouse also needs time to adjust." 4. "You sound unhappy. Have you tried to talk to your spouse?"

"You sound unhappy. Have you tried to talk to your spouse?" The response "You sound unhappy. Have you tried to talk to your spouse?" identifies the client's feelings and accepts them but also points out the responsibility of the client to take action. Although the response "You don't get along with your spouse" identifies one of the client's concerns, the identification of the underlying feeling is more therapeutic. The response "I'm sorry. What can I do to make you feel better?" makes the nurse responsible for changing the situation, which is not appropriate or therapeutic. The response "It may be temporary because your spouse also needs time to adjust" denies the client's feelings and provides false reassurance.

The RN is caring for a patient with diabetes admitted with hypoglycemia that occurred at home. Which teaching points for treatment of hypoglycemia at home would the nurse include in a teaching plan for the patient and family before discharge? Select all that apply. 1. Signs and symptoms of hypoglycemia include hunger, irritability, weakness, headache, and blood glucose less than 60 mg/dL (3.3 mmol/L). 2. Treat hypoglycemia with 4 to 8 g of carbohydrate such as glucose tablets or ¼ cup (60 mL) of fruit juice. 3. Retest blood glucose in 30 minutes. 4. Repeat the carbohydrate treatment if the symptoms do not resolve. 5. Eat a small snack of carbohydrate and protein if the next meal is more than an hour away. 6. If the patient has severe hypoglycemia, does not respond to treatment, and is unconscious, transport to the emergency department (ED).

1,4,5,6 The manifestations listed in option 1 are correct. The symptoms should be treated with carbohydrate, but 10 to 15 g (not 4 to 8 g). Glucose should be retested at 15 minutes; 30 minutes is too long to wait. Options 4 and 5 are correct. When a patient has severe hypoglycemia, does not respond to administration of glucagon, and remains unconscious, he or she should be transported to the ED and the health care provider notified.

A patient with newly diagnosed diabetes has peripheral neuropathy. Which key points should the nurse include in the teaching plan for this patient? Select all that apply. 1. "Clean and inspect your feet every day." 2. "Be sure that your shoes fit properly." 3. "Nylon socks are best to prevent friction on your toes from shoes." 4. "Only a podiatrist should trim your toenails." 5. "Report any nonhealing skin breaks to your health care provider (HCP)." 6. "Use a thermometer to check the temperature of water before taking a bath."

1. "Clean and inspect your feet every day." 2. "Be sure that your shoes fit properly." 5. "Report any nonhealing skin breaks to your health care provider (HCP)." 6. "Use a thermometer to check the temperature of water before taking a bath." Sensory alterations are the major cause of foot complications in patient with diabetes, and patients should be taught to examine their feet on a daily basis. Properly fitted shoes protect the patient from foot complications. Broken skin increases the risk of infection. Cotton socks are recommended to absorb moisture. Using a bath thermometer can prevent burn injuries. Patients, family, or HCPs may trim toenails.

Which statement by a client with type 2 diabetes indicates to the nurse that additional dietary teaching is needed? 1. "I can eat as much dietetic fruit as I want." 2. "I can have a lettuce salad whenever I want it." 3. "I know that half of my diet should be carbohydrates." 4. "I need to reduce the amounts of saturated fats in my diet."

1. "I can eat as much dietetic fruit as I want." The client needs further teaching; dietetic fruit is not sugar free and must be calculated in a diabetic individual's diet. Lettuce is considered a free food in the diet of a diabetic person. It is suggested that the caloric intake of a diabetic person's diet should be 50% carbohydrate, 20% protein, and 30% fat. Saturated fats should be limited to 10% of the fat intake; 90% of fat should be unsaturated fats.

The RN is orienting a new graduate nurse who is providing diabetes education for a patient about insulin injection. For which teaching statement by the new nurse must the RN intervene? 1. "To prevent lipohypertrophy, be sure to rotate injection sites from the abdomen to the thighs." 2. "To correctly inject the insulin, lightly grasp a fold of skin and inject at a 90-degree angle." 3. "Always draw your regular insulin into the syringe first before your NPH (neutral protamine Hagedorn) insulin." 4. "Avoid injecting the insulin into scarred sites because those areas slow the absorption rate of insulin."

1. "To prevent lipohypertrophy, be sure to rotate injection sites from the abdomen to the thighs." Although it is important to rotate injection sites for insulin, it is preferred that the injection sites be rotated within one anatomic site (e.g., the abdomen) to prevent day-to-day changes in the absorption rate of the insulin. All of the other teaching points are appropriate.

The nurse is caring for a patient with diabetes who is developing diabetic ketoacidosis (DKA). Which task delegation or assignment is most appropriate? 1. Ask the unit clerk to page the health care provider to come to the unit. 2. Ask the LPN/LVN to administer IV push insulin according to a sliding scale. 3. Ask the unlicensed assistive personnel (UAP) to hang a new bag of normal saline. 4. Ask the UAP to get the patient a cup (236 mL) of orange juice.

1. Ask the unit clerk to page the health care provider to come to the unit. The nurse should not leave the patient. The scope of the unit clerk's job includes calling and paging physicians. LPNs/LVNs generally do not administer IV push medication, although in some states with additional training, this may be done. (Be sure to check the Scope of Practice in your specific state.) IV fluid administration is not within the scope of practice of UAPs. Patients with DKA already have a high glucose level and do not need orange juice.

The nurse is caring for an older patient with type 1 diabetes and diabetic retinopathy. What is the nurse's priority concern for assessing this patient? 1. Assess ability to measure and inject insulin and to monitor blood glucose levels. 2. Assess for damage to motor fibers, which can result in muscle weakness. 3. Assess which modifiable risk factors can be reduced. 4. Assess for albuminuria, which may indicate kidney disease.

1. Assess ability to measure and inject insulin and to monitor blood glucose levels. The older patient with diabetic retinopathy also has general age-related vision changes, and the ability to perform self-care may be seriously affected. He or she may have blurred vision, distorted central vision, fluctuating vision, loss of color perception, and mobility problems resulting from loss of depth perception. When a patient has visual changes, it is especially important to assess his or her ability to measure and inject insulin and to monitor blood glucose levels to determine if adaptive devices are needed to assist in self-management. The other options are important but are not specific to diabetic retinopathy.

The unlicensed assistive personnel reports to the RN that a patient with type 1 diabetes has a question about exercise. What important points would the RN be sure to teach this patient? Select all that apply. 1. Exercise guidelines are based on blood glucose and urine ketone levels. 2. Be sure to test your blood glucose only after exercising. 3. You can exercise vigorously if your blood glucose is between 100 and 250 mg/dL (5.6 and 13.9 mmol/L). 4. Exercise will help resolve the presence of ketones in your urine. 5. A 5- to 10-minute warm-up and cool-down period should be included in your exercise. 6. For unplanned exercise, increased intake of carbohydrates is usually needed.

1. Exercise guidelines are based on blood glucose and urine ketone levels. 3. You can exercise vigorously if your blood glucose is between 100 and 250 mg/dL (5.6 and 13.9 mmol/L). 5. A 5- to 10-minute warm-up and cool-down period should be included in your exercise. 6. For unplanned exercise, increased intake of carbohydrates is usually needed. Guidelines for exercise are based on blood glucose and urine ketone levels. Patients should test blood glucose before, during, and after exercise to be sure that it is safe to exercise. When ketones are present in urine, the patient should not exercise because ketones indicate that current insulin levels are not adequate. Vigorous exercise is permitted in patients with type 1 diabetes if glucose levels are between 100 and 250 mg/dL (5.6 and 13.9 mmol/L). Warm-up and cool-down should be included in exercise to gradually increase and decrease the heart rate. For planned exercise, reduction in insulin dosage is used for hypoglycemia prevention. For unplanned exercise, intake of additional carbohydrate is usually needed.

The nurse develops a teaching plan for a client with diabetes who has been diagnosed with lower extremity arterial disease (LEAD). Which measures would the nurse include to increase arterial blood flow to the extremities? 1. Exercises that promote muscular activity 2. Meticulous care of minor skin breakdown 3. Elevation of the legs above the level of the heart 4. Soaking the feet in hot water each day

1. Exercises that promote muscular activity Arterial blood flow is improved with exercise by fostering the development of collateral circulation. Meticulous care of minor skin breakdown is important for the person with diabetes, but it does not improve arterial blood flow. Elevating the legs above the heart reduces arterial blood flow; the legs should be kept dependent to facilitate tissue perfusion. Soaking the feet in hot water is contraindicated because it can burn the skin or cause drying; also, individuals with diabetes may have neuropathies, which alter the perception of temperature.

A 58-year-old patient with type 2 diabetes was admitted to the acute care unit with a diagnosis of chronic obstructive pulmonary disease (COPD) exacerbation. When the RN prepares a care plan for this patient, what would he or she be sure to include? Select all that apply. 1. Fingerstick blood glucose checks before meals and at bedtime 2. Sliding-scale insulin dosing as prescribed 3. Bed rest until the COPD exacerbation is resolved 4. Teaching about the Atkins diet for weight loss 5. Demonstration of the components of foot care 6. Discussing the relationship between illness and glucose levels

1. Fingerstick blood glucose checks before meals and at bedtime 2. Sliding-scale insulin dosing as prescribed 5. Demonstration of the components of foot care 6. Discussing the relationship between illness and glucose levels When a patient with diabetes is ill, glucose levels become elevated, and administration of insulin may be necessary. Administration of sliding-scale insulin is guided by fingerstick blood glucose checks. Teaching or reviewing the components of proper foot care is always a good idea with a patient with diabetes. Bed rest is not necessary, and glucose levels may be better controlled when a patient is more active. The Atkins diet recommends decreasing the consumption of carbohydrates and is not a good diet for patient with diabetes.

An LPN/LVN is assigned to administer rapid-acting insulin, lispro, to a patient with type 1 diabetes. What essential information would the RN be sure to tell the LPN/LVN? 1. Give this insulin when the food tray has been delivered and the patient is ready to eat. 2. Only give this insulin for fingerstick glucose reading is above 200 mg/dL (11.1 mmol/L). 3. This insulin mimics the basal glucose control of the pancreas. 4. Rapid-acting insulin is the only insulin that can be given subcutaneously or IV.

1. Give this insulin when the food tray has been delivered and the patient is ready to eat. The onset of action for rapid-acting insulin is within minutes, so it should be given only when the patient has food and is ready to eat. Because of this, rapid-acting insulin is sometimes called "see food" insulin. Options 2, 3, and 4 are incorrect with regard to rapid-acting insulin. Option 2 is incorrect with regard to all forms of insulin. Long-acting insulins mimic the action of the pancreas. Regular insulin is the only insulin that can be given IV.

The nurse is caring for an 81-year-old adult with type 2 diabetes, hypertension, and peripheral vascular disease. Which admission assessment findings increase the patient's risk for development of hyperglycemic-hyperosmolar syndrome (HHS)? Select all that apply. 1. Hydrochlorothiazide (HCTZ) prescribed to control her blood pressure 2. Weight gain of 6 lb (2.7 kg) over the past month 3. Avoids consuming liquids in the evening 4. Blood pressure of 168/94 mm Hg 5. Urine output of 50 to 75 mL/hr 6. Glucose greater than 600 mg/dL (33.3 mmol/L)

1. Hydrochlorothiazide (HCTZ) prescribed to control her blood pressure 3. Avoids consuming liquids in the evening 6. Glucose greater than 600 mg/dL (33.3 mmol/L) HHS often occurs in older adults with type 2 diabetes. Risk factors include taking diuretics and inadequate fluid intake. Serum glucose is greater than 600 mg/dL (33.3 mmol/L). Weight loss (not weight gain) would be a symptom. Although the patient's blood pressure is high, this is not a risk factor. A urine output of 50 to 75 mL/hr is adequate.

An adolescent is found to have type 1 diabetes. The nurse plans to teach the adolescent that dietary control and exercise can help regulate the disorder. Which additional information would the nurse include in the teaching plan? Select all that apply. One, some, or all responses may be correct. 1. Insulin therapy 2. Prophylactic antibiotics 3. Blood glucose monitoring 4.Oral hypoglycemic agents 5. Adherence to the treatment regimen

1. Insulin therapy 3. Blood glucose monitoring 5. Adherence to the treatment regimen Because clients with type 1 diabetes have little or no endogenous insulin, they must take insulin. Blood glucose monitoring is an important aspect of therapy because it aids evaluation of the effectiveness of diabetic control. Dietary control and exercise reduce the amount of exogenous insulin needed. Although adhering to the diabetic regimen is difficult, especially for adolescents who need to identify with their peers, its importance in promoting euglycemia should be discussed. Although infection increases insulin requirements, prophylactic antibiotics are not needed. Oral hypoglycemics are ineffective in stimulating insulin secretion in clients with type 1 diabetes.

An increase in which blood component is responsible for the acidosis related to untreated diabetes mellitus? 1. Ketones 2. Glucose 3. Lactic acid 4. Glutamic acid

1. Ketones The ketones produced excessively in diabetes are a byproduct of the breakdown of body fats and proteins for energy; this occurs when insulin is not secreted or is unable to be used to transport glucose across the cell membrane into the cells. The major ketone, acetoacetic acid, is an alpha-ketoacid that lowers the blood pH, resulting in acidosis. Glucose does not change the pH. Lactic acid is produced as a result of muscle contraction; it is not unique to diabetes. Glutamic acid is a product of protein metabolism.

A client with type 1 diabetes for 25 years states, "I have been really bad for the past 15 years. I have not paid attention to my diet and have done little to control my diabetes." Which common complications of diabetes might the nurse expect to identify when assessing this client? Select all that apply. One, some, or all responses may be correct. 1. Leg ulcers 2. Loss of visual acuity 3. Thick, yellow toenails 4. Increased growth of body hair 5. Decreased sensation in the feet

1. Leg ulcers 2. Loss of visual acuity 3. Thick, yellow toenails 5. Decreased sensation in the feet Leg ulcers are a common response to the microvascular and macrovascular changes associated with diabetes. Retinopathy, damage to the microvascular system of the retina (e.g., edema, exudate, and local hemorrhage), occurs as a result of the occlusion of the small vessels in the eyes, causing microaneurysms in the capillary walls. Thick, yellow toenails result from prolonged inadequate arterial circulation to the feet. Pedal pulses diminish, which can result in gangrene, necessitating amputation. Diabetic neuropathies affect 60% to 70% of people with diabetes. It is theorized that consistent hyperglycemia causes a buildup of sorbitol and fructose in the nerves that results in impairment via an unknown process. Inadequate arterial circulation to hair follicles results in a lack of hair on the feet and ankles. The skin becomes dry and cracks, predisposing it to leg ulcers and infection.

Which common complications of diabetes would the nurse assess for in a client with a long history of the disease? Select all that apply. One, some, or all responses may be correct. 1. Leg ulcers 2. Loss of visual acuity 3. Increased creatinine clearance 4. Prolonged capillary refill in the toes 5. Decreased sensation in the lower extremities

1. Leg ulcers 2. Loss of visual acuity 4. Prolonged capillary refill in the toes 5. Decreased sensation in the lower extremities Ulcers of the legs are a common response to the microvascular and macrovascular changes associated with diabetes. Retinopathy, damage to the microvascular system of the retina (e.g., edema, exudate, and local hemorrhage), occurs as a result of occlusion of the small vessels, causing microaneurysms in the capillary walls. Macrovascular changes in the distal capillary beds interfere with blood flow to the distal extremities. Decreased sensation in the lower extremities is a complication of diabetes. Consistent hyperglycemia causes a buildup of sorbitol and fructose in the nerves that leads to impairment via an unknown process. Creatinine clearance decreases, not increases, as renal function deteriorates in response to microvascular damage to the small blood vessels that supply the glomeruli.

The nurse is teaching an adolescent newly diagnosed with type 1 diabetes about self-care. Which is the primary long-term goal? 1. Maintaining normoglycemia 2. Complying with the diabetic diet 3. Adhering to an exercise program 4. Developing a nonstressful lifestyle

1. Maintaining normoglycemia Maintaining normoglycemia is a realistic goal because it decreases the risk of complications such as neuropathy, retinopathy, and atherosclerosis. A regimen of insulin, exercise, and diet will help the adolescent achieve this goal. Compliance with a diabetic diet is an objective because it will help the adolescent achieve the long-term goal; diet alone is insufficient to achieve normoglycemia. Adherence to an exercise program is an objective because it will help the adolescent achieve the long-term goal; exercise alone is insufficient to achieve normoglycemia. Development of a lower stress lifestyle is a worthwhile goal, but developing a nonstressful lifestyle is not realistic.

A client who is 60 pounds (27.2 kg) more than the ideal body weight is admitted to the hospital with a diagnosis of type 1 diabetes. Which concept would the nurse include in teaching about diabetes when discussing strategies to lose weight? 1. Obesity leads to insulin resistance. 2. Surplus fat causes excretion of insulin. 3. Fat cells absorb insulin and prevent its circulation to other cells. 4. Lipids accumulate in the pancreas and interfere with insulin production.

1. Obesity leads to insulin resistance. Excess fat alters glucose metabolism, causing cells to become insulin resistant. Fat cells have no relationship to the function of the kidneys. Fat cells do not absorb insulin and therefore do not prevent the circulation of insulin to other cells. Clients with type 1 diabetes do not produce insulin. If lipids should accumulate in the pancreas of a healthy adult, they do not interfere with insulin production.

The nurse determines that an adolescent with newly diagnosed type 1 diabetes has sufficient knowledge of the disorder. Which is the next appropriate action? 1. Setting goals with the client 2. Developing a rapport with the client 3. Teaching the client how to give insulin injections 4. Instructing the client how to monitor blood glucose

1. Setting goals with the client A negotiation of goals is essential to successful learning; mutual goal-setting provides a focus for learning. A rapport should have developed before teaching of the adolescent about diabetes was started. Teaching the client how to give injections or monitor the blood glucose level is premature. If the client does not identify a specific need or set a goal, motivation may be minimal.

12. A patient newly diagnosed with diabetes is about to be discharged home. You are watching the patient administer insulin. Which of the following actions causes you to re-educate them? A. They massaged the site after administering the insulin. B. They injected into the fat of their thighs. C. They used an opposite side for injection compared to the last insulin injection. D. They engaged the safety after administering the medication.

12. A.

The RN is the preceptor for a senior nursing student who will teach a patient with diabetes about self-care during sick days. For which statement by the student must the RN intervene? 1. "When you are sick, be sure to monitor your blood glucose at least every 4 hours." 2. "Test your urine for ketones whenever your blood glucose level is less than 240 mg/dL (13.3 mmol/L)." 3. "To prevent dehydration, drink 8 ounces (236 mL) of sugar-free liquid every hour while you are awake." 4. "Continue to eat your meals and snacks at the usual times."

2. "Test your urine for ketones whenever your blood glucose level is less than 240 mg/dL (13.3 mmol/L)." Urine ketone testing should be done whenever the patient's blood glucose is greater than 240 mg/dL (13.3 mmol/L). All of the other teaching points are appropriate "sick day rules." For dehydration, teaching should also include that if the patient's blood glucose is lower than her target range, she should drink fluids containing sugar.

An adolescent was recently diagnosed with type 2 diabetes mellitus. Which information will the nurse include when providing education to the family? 1. "Your teen will need insulin injections for the rest of her life." 2. "The most important interventions are good nutrition and portion control." 3. "This is a condition where the body produces antibodies against its own cells." 4. "This condition causes weight loss and increased appetite, thirst, and urination."

2. "The most important interventions are good nutrition and portion control." Most children with type 2 diabetes are overweight or at risk for becoming overweight. With nutritional intervention to promote proper weight, the condition may often be managed with diet and exercise alone. A lifelong insulin regimen, the production of antibodies against the child's own cells, and weight loss with increased appetite, thirst, and urination are all typical of type 1 diabetes.

While the RN is performing an admission assessment on a patient with type 2 diabetes, the patient states that he routinely drinks 3 beers a day. What is the nurse's priority follow-up question at this time? 1. "Do you have any days when you do not drink?" 2. "When during the day do you drink your beers?" 3. "Do you drink any other forms of alcohol?" 4 . "Have you ever had a lipid profile completed?"

2. "When during the day do you drink your beers?" Alcohol has the potential for causing alcohol-induced hypoglycemia. It is important to know when the patient drinks alcohol and to teach the patient to ingest it shortly after meals to prevent this complication. The other questions are important but not urgent. The lipid profile question is important because alcohol can raise plasma triglycerides but is not as urgent as the potential for hypoglycemia.

The RN is serving as preceptor to a new graduate nurse who has recently passed the RN licensure (NCLEX®) examination. The new nurse has only been on the unit for 2 days. Which patient should be assigned to the new graduate nurse? 1. A 68-year-old patient with diabetes who is showing signs of hyperglycemia 2. A 58-year-old patient with diabetes who has cellulitis of the left ankle 3. A 49-year-old patient with diabetes just returned from the postanesthesia care unit after a below-knee amputation 4. A 72-year-old patient with diabetes with diabetic ketoacidosis who is receiving IV insulin

2. A 58-year-old patient with diabetes who has cellulitis of the left ankle The new nurse is very early in orientation to the unit. Appropriate patient assignments at this time include patients whose conditions are stable and not complex. Patients 1, 3, and 4 are more complex and will benefit from care by a nurse experienced in care of patients with diabetes.

Which actions can the school nurse delegate to an experienced unlicensed assistive personnel (UAP) who is working with a 7-year-old child with type 1 diabetes in an elementary school? Select all that apply. 1. Obtaining information about the child's usual insulin use from the parents 2. Administering oral glucose tablets when blood glucose level falls below 60 mg/dL (3.3 mmol/L) 3. Teaching the child about what foods have high carbohydrate levels 4. Obtaining blood glucose readings using the child's blood glucose monitor 5. Reminding the child to have a snack after the physical education class 6. Assessing the child's knowledge level about his or her type 1 diabetes

2. Administering oral glucose tablets when blood glucose level falls below 60 mg/dL (3.3 mmol/L) 4. Obtaining blood glucose readings using the child's blood glucose monitor 5. Reminding the child to have a snack after the physical education class National guidelines published by the American Diabetes Association (ADA) indicate that administration of emergency treatment for hypoglycemia (e.g., glucose tablets), obtaining blood glucose readings, and reminding children about content they have already been taught by licensed caregivers are appropriate tasks for non-health care professional personnel such as teachers, paraprofessionals, and UAP. Assessments and education require more specialized education and scope of practice and should be done by the school nurse.

2. A patient with diabetes asks you about what type of exercise they should perform throughout the week. The best response is: A. Lifting weights B. Sprinting C. Swimming D. Jumping

2. C

A patient with diabetes has hot, dry skin; rapid and deep respirations; and a fruity odor to his breath. The charge nurse observes a newly graduated RN performing all the following patient tasks. Which action requires that the charge nurse intervene immediately? 1. Checking the patient's fingerstick glucose level 2. Encouraging the patient to drink orange juice 3. Checking the patient's order for sliding-scale insulin dosing 4. Assessing the patient's vital signs every 15 minutes

2. Encouraging the patient to drink orange juice The signs and symptoms the patient is exhibiting are consistent with hyperglycemia. The RN should not give the patient additional glucose. All of the other interventions are appropriate for this patient. The RN should also notify the health care provider at this time.

The nurse is responsible for the care of a patient with diabetes who is unable to swallow, is unconscious and seizing, and has a blood glucose of less than 20 mg/dL (1.1 mmol/L). Which actions are most appropriate responses for this patient at this time? Select all that apply. 1. Check the chart for the patient's most recent A1c level. 2. Give glucagon 1 mg subcutaneously or intramuscularly (IM). 3. Repeat the dose of glucagon in 10 minutes if the patient remains unconscious. 4. Apply aspiration precautions because glucagon can cause vomiting. 5. Give the patient an oral simple sugar or snack. 6. Notify the health care provider (HCP) immediately

2. Give glucagon 1 mg subcutaneously or intramuscularly (IM). 3. Repeat the dose of glucagon in 10 minutes if the patient remains unconscious. 4. Apply aspiration precautions because glucagon can cause vomiting. 6. Notify the health care provider (HCP) immediately This patient's manifestations suggest severe hypoglycemia. Essential actions at this time include notifying the HCP immediately and giving glucagon 1 mg subcutaneously or IM. Glucagon is the main counterregulatory hormone to insulin and is used as first-line therapy for severe hypoglycemia in patients with diabetes. The dose or glucagon is repeated after 10 minutes if the patient remains unconscious. Aspiration precautions are important because this drug can cause vomiting. Checking the patient's A1c level is not important at this time. Offering oral glucose or a snack when a patient is unable to swallow or unconscious is inappropriate.

A student with type 1 diabetes asks the nurse which hormone causes the blood glucose level to rise. Which hormone would the nurse report? 1. Insulin 2. Glucagon 3. Epinephrine 4. Adrenocorticotropic hormone (ACTH)

2. Glucagon Glucagon promotes liver glycogenolysis, resulting in the release of glucose into the blood. ACTH is not directly related to glycogenolysis; it is released from the anterior pituitary. Insulin production is not directly related to glycogenolysis; in healthy individuals the level of insulin will increase as the glucose level increases. Epinephrine is not directly related to glycogenolysis; it is released from the adrenal medulla and sympathetic nerve endings.

Which symptoms would be seen in a client with type 1 diabetes who is experiencing hypoglycemia? Select all that apply. One, some, or all responses may be correct. 1. Vomiting 2. Headache 3. Tachycardia 4. Cool, clammy skin 5. Increased respirations

2. Headache 3. Tachycardia 4. Cool, clammy skin Headache is a neuroglycopenic response directly related to brain glucose deprivation. Tachycardia occurs with hypoglycemia because of a neurogenic adrenergic response; it is a sympathetic nervous system response precipitated by a low blood glucose level. Cool, clammy skin is a neurogenic cholinergic response; it is a sympathetic nervous system response precipitated by a low serum glucose level. Vomiting occurs with hyperglycemia because of the effects of metabolic acidosis. Increased respirations are a sign of hyperglycemia and are related to metabolic acidosis; this is a compensatory response in an attempt to blow off carbon dioxide and increase the pH level.

Which would the nurse emphasize when teaching lifelong management of type 1 diabetes to an adolescent? 1. Soaking the feet in hot water each day 2. Inspecting both feet frequently for signs of trauma 3. Drying the feet thoroughly after a bath by rubbing with a towel 4. Treating minor cuts on the feet with an antiseptic such as iodine

2. Inspecting both feet frequently for signs of trauma Adequate inspection of the feet should become a habit; it is the quickest and easiest means of identifying pressure sites and preventing infection. Hot water should never be used, because it may cause burn injury of the skin. The feet should be patted dry, not rubbed; rubbing may cause abrasions and injure the skin. Strong antiseptics are too harsh and should not be used because they may cause injury to the skin.

When obtaining a health history from a client recently diagnosed with type 1 diabetes, the nurse expects the client to report which clinical manifestation? 1. Nervousness 2. Polyuria 3. Nocturia 4. Diaphoresis

2. Polyuria Excessive thirst (polydipsia), excessive hunger (polyphagia), and frequent urination (polyuria) are caused by the body's inability to metabolize glucose adequately. Lethargy, not nervousness, occurs because of a lack of metabolized glucose for energy. Frequent urination occurs throughout a 24-hour period because glucose in the urine pulls fluid with it. Diaphoresis occurs with severe hypoglycemia, not hyperglycemia.

Which common cause of diabetic ketoacidosis would the nurse consider when caring for a postoperative client with diabetes? 1. Emotional stress 2. Presence of infection 3. Increased insulin dose 4. Inadequate food intake

2. Presence of infection Infection increases the body's metabolic rate, and insulin is not available for increased demands. Although emotional stress will affect glucose levels, diabetic ketoacidosis will rarely result. Increased insulin dose will lead to insulin coma (hypoglycemia) if diet is not increased as well. Inadequate food intake will result in insulin coma.

In the emergency department, during initial assessment of a newly admitted patient with diabetes, the nurse discovers all of these findings. Which finding should be reported to the health care provider immediately? 1. Hammer toe of the left second metatarsophalangeal joint 2. Rapid respiratory rate with deep inspirations 3. Numbness and tingling bilaterally in the feet and hands 4. Decreased sensitivity and swelling of the abdomen

2. Rapid respiratory rate with deep inspirations Rapid, deep respirations (Kussmaul respirations) are symptomatic of diabetic ketoacidosis. Hammer toe, as well as numbness and tingling, are chronic complications associated with diabetes. Decreased sensitivity and swelling (lipohypertrophy) occur at a site of repeated insulin injections, and treatment involves teaching the patient to rotate injection sites within one anatomic site.

An older patient with type 2 diabetes has cardiovascular autonomic neuropathy (CAN). Which instruction would the nurse provide for the unlicensed assistive personnel (UAP) assisting the patient with morning care? 1. Provide a complete bed bath for this patient. 2. Sit the patient up slowly on the side of the bed before standing. 3. Only let the patient wash his or her face and brush his or her teeth. 4. Be sure to provide rest periods between activities.

2. Sit the patient up slowly on the side of the bed before standing. CAN affects sympathetic and parasympathetic nerves of the heart and blood vessels. It may lead to orthostatic (postural) hypotension and syncope (brief loss of consciousness on standing) caused by failure of the heart and arteries to respond to position changes by increasing heart rate and vascular tone. The nurse should be sure to instruct the UAP to have the patient change positions slowly when moving from lying to sitting and standing.

Which outcome is the best indication that a client with type 1 diabetes is successfully managing the disease? 1. Reduction in excess body weight 2. Stabilization of the serum glucose 3. Demonstrated knowledge of the disease 4. Adherence to the prescription for insulin

2. Stabilization of the serum glucose A combination of diet, exercise, and medication is necessary to control the disease; the interaction of these therapies is reflected by the serum glucose level. Weight loss may occur with inadequate insulin. Acquisition of knowledge does not guarantee its application. Insulin alone is not enough to control the disease.

An adolescent who has had type 1 diabetes for 5 years stops adhering to the therapeutic regimen. Which would the nurse conclude based on the client's developmental level? 1. Need for attention 2. Struggle for identity 3. Denial of the diabetes 4. Regression related to the illness

2. Struggle for identity Striving to attain identity and independence are tasks of the adolescent, and rebellion against established norms may be exhibited. Nonadherence to a regimen is not a bid for attention; rather, it is an attempt to establish an identity, which is a developmental task of adolescence. Although the adolescent may be using denial, denial is not developmentally related to adolescence. Noncompliance is not a sign of regression; it is an attempt to attain identity through rebellion against established norms.

An adolescent who has just been found to have type 1 diabetes asks the nurse about exercise. Which is the best response by the nurse? 1. "Exercise should be restricted." 2. "Exercise will increase blood glucose." 3. "Extra snacks are needed before exercise." 4. "Extra insulin is required during exercise."

3. "Extra snacks are needed before exercise." Exercise lowers the blood glucose level; an extra snack can prevent hypoglycemia. Exercise is encouraged, not restricted. Exercise lowers, not increases, blood glucose. Extra insulin is contraindicated because exercise decreases the blood glucose level; extra insulin may precipitate hypoglycemia.

The patient with type 2 diabetes has a health care provider prescription for NPO status for a cardiac catheterization. An LPN/LVN who is assigned to administer medications to this patient asks the supervising RN whether the patient should receive his ordered repaglinide. What is the RN's best response? 1. "Yes, because this drug will increase the patient's insulin secretion and prevent hyperglycemia." 2. "No, because this drug may cause the patient to experience gastrointestinal symptoms such as nausea." 3. "No, because this drug should be given 1 to 30 minutes before meals and the patient is NPO." 4. "Yes, because this drug should be taken three times a day whether the patient eats or not."

3. "No, because this drug should be given 1 to 30 minutes before meals and the patient is NPO." Repaglinide is a meglitinide analog drug. These drugs are short-acting agents used to prevent postmeal blood glucose elevation. They should be given within 1 to 30 minutes before meals and cause hypoglycemia shortly after dosing when a meal is delayed or omitted.

When teaching an adolescent with type 1 diabetes about dietary management, which instruction would the nurse include? 1. Meals should be eaten at home. 2. Foods should be weighed on a gram scale. 3. A ready source of glucose should be available. 4. Specific foods should be cooked for the adolescent.

3. A ready source of glucose should be available. An adolescent with type 1 diabetes must carry a source of simple sugar (e.g., glucose tablets, Insta-Glucose, sugar-containing candy such as LifeSavers) to rapidly counteract the effects of hypoglycemia. This should be followed by a complex carbohydrate and a protein. Stating that meals should be eaten at home is an unrealistic and unnatural instruction for an adolescent. Stating that foods should be weighed on a gram scale is an unnecessary and time-consuming procedure. The adolescent should be made to feel a part of the family; the recommended diet is nutritious and no different from that of the rest of the family.

A patient has newly diagnosed type 2 diabetes. Which action should the RN assign to an LPN/LVN rather than an experienced unlicensed assistive personnel (UAP)? 1. Measuring the patient's vital signs every shift 2. Checking the patient's glucose level before each meal 3. Administering subcutaneous insulin on a sliding scale as needed 4. Assisting the patient with morning care

3. Administering subcutaneous insulin on a sliding scale as needed The UAP's scope of practice includes checking vital signs and assisting with morning care. Experienced UAPs with special training can check the patient's glucose level before meals and at bedtime. It is generally not within the UAP's scope of practice to administer medications, but this is within the scope of practice of the LPN/LVN.

The nurse is helping an adolescent with type 1 diabetes establish a consistent meal pattern. Which feedback indicates that further teaching is needed? 1. Weighs portion sizes for several months 2. Reads nutrition labels on prepared foods 3. Avoids complex carbohydrate substitutes 4. Limits sugar alternatives containing sorbitol

3. Avoids complex carbohydrate substitutes Complex carbohydrates may be substituted, depending on caloric content and amount eaten per serving. Flexibility is needed to promote adherence to any dietary regimen. Using consistent portion sizes is a key to maintaining diabetic control. By weighing and measuring portion sizes for several months the adolescent learns to recognize the acceptable amount to be eaten at a glance. The adolescent should read nutrition labels carefully, especially for their carbohydrate and caloric content. Most dietetic foods contain sorbitol. Sorbitol metabolizes to fructose and then glucose, so its use should be restricted when possible.

3. Which patient is MOST likely to develop Diabetic Ketoacidosis? A. A 25 year old female newly diagnosed with Cushing's Disease taking glucocorticoids. B. A 36 year old male with diabetes mellitus who has been unable to eat the past 2 days due to a gastrointestinal illness and has not been taking insulin. C. A 35 year old female newly diagnosed with Type 2 diabetes. D. None of the options are correct.

3. B

While obtaining the client's health history, which factor would the nurse identify that predisposes the client to type 2 diabetes? 1. Having diabetes insipidus 2. Eating low-cholesterol foods 3. Being 20 pounds (9 kg) overweight 4. Drinking a daily alcoholic beverage

3. Being 20 pounds (9 kg) overweight Excessive body weight is a known predisposing factor to type 2 diabetes; the exact relationship is unknown. Diabetes insipidus is caused by too little antidiuretic hormone (ADH) and has no relationship to type 2 diabetes. High-cholesterol diets and atherosclerotic heart disease are associated with type 2 diabetes. Alcohol intake is not known to predispose a person to type 2 diabetes.

The nurse is planning to teach an adolescent about diabetes and self-administration of insulin. Which would the nurse complete first? 1. Establish realistic goals. 2. Assess the adolescent's intellectual ability. 3. Determine what the adolescent knows about diabetes. 4. Gather the equipment that will be needed for the demonstration

3. Determine what the adolescent knows about diabetes. Before developing and instituting a teaching plan, the nurse must assess the adolescent's attitudes, experience, knowledge, and understanding of the health problem. Before realistic goals can be set there must be an assessment. The adolescent's intellectual ability is only one aspect of the information the nurse must collect and can be assessed as the nurse is determining what the adolescent knows about diabetes. Performing a demonstration is premature until readiness for learning has been established.

Which clinical findings would cause the nurse to suspect that an adolescent child with type 1 diabetes is hypoglycemic? 1. Increased thirst, sleepiness, and nausea 2. Confusion, dry mouth, and diminished reflexes 3. Difficulty concentrating, hunger, and diaphoresis 4. Flushed face, deep breathing, and abdominal pain

3. Difficulty concentrating, hunger, and diaphoresis Difficulty concentrating, hunger, and diaphoresis are the most common signs and symptoms of hypoglycemia. Increased adrenergic nervous system activity and increased catecholamine secretion produce hunger and diaphoresis. Difficulty concentrating reflects central nervous system glucose deprivation. Increased thirst, sleepiness, and nausea are signs and symptoms of hyperglycemia as ketoacidosis develops. Confusion, dry mouth, and diminished reflexes are signs and symptoms of hyperglycemia; they reflect ketoacidosis. Flushed face, deep breathing, and abdominal pain are signs and symptoms of ketoacidosis.

The nurse is preparing to review a teaching plan for a patient with type 2 diabetes mellitus. To determine the patient's level of compliance with his prescribed diabetic regimen, which value would the nurse be sure to review? 1. Fasting glucose level 2. Oral glucose tolerance test results 3. Glycosylated hemoglobin (HgbA1c) level 4. Fingerstick glucose findings for 24 hours

3. Glycosylated hemoglobin (HgbA1c) level The higher the blood glucose level is over time, the more glycosylated the hemoglobin becomes. The HgbA1c level is a good indicator of the average blood glucose level over the previous 120 days. Fasting glucose and oral glucose tolerance tests are important diagnostic tools. Fingerstick blood glucose monitoring provides information that allows adjustment of the patient's therapeutic regimen.

Which laboratory value supports the presence of diabetic ketoacidosis in a client with type 1 diabetes? 1. Decreased serum glucose levels 2. Decreased serum calcium levels 3. Increased blood urea nitrogen levels 4. Increased serum bicarbonate levels

3. Increased blood urea nitrogen levels With diabetic ketoacidosis, blood urea nitrogen level generally is increased because of dehydration. With diabetic ketoacidosis, the serum glucose levels are generally greater than 300 mg/dL (16.7 mmol/L). The calcium level is unrelated to diabetic ketoacidosis. Serum bicarbonate levels are less than 15 mEq/L (15 mmol/L).

Which increased physiological response would the nurse include when explaining the need for weight loss to a client who is diagnosed with diabetes? 1. Fatty acid storage 2. Glucose oxidation 3. Insulin requirements 4. Cellular entry of glucose

3. Insulin requirements Obesity causes insulin resistance at the cellular level, so more insulin is required for transfer of glucose across cell membranes. Fatty acid metabolism is altered. Fatty acids break down; storage decreases. With obesity, oxidation of glucose decreases and insulin needs increase. Obesity causes peripheral cellular resistance to glucose entry into cells.

Which fluid shift will the nurse take into consideration when assessing a client with type 1 diabetes who is experiencing a fluid imbalance? 1. Intravascular to interstitial as a result of glycosuria 2. Extracellular to interstitial as a result of hypoproteinemia 3. Intracellular to intravascular as a result of hyperosmolarity 4. Intercellular to intravascular as a result of increased hydrostatic pressure

3. Intracellular to intravascular as a result of hyperosmolarity The osmotic effect of hyperglycemia pulls fluid from the cells, resulting in cellular dehydration. Hyperglycemia pulls fluid from the interstitial compartment to the intravascular compartment. Interstitial fluid is part of the extracellular compartment; the osmotic pull of glucose exceeds that of other osmotic forces. An increase in hydrostatic pressure results in an intravascular-to-interstitial shift.

Which is the primary fluid shift that occurs with diabetes mellitus? 1. Intravascular to interstitial because of glycosuria 2. Interstitial to extracellular because of hypoproteinemia 3. Intracellular to intravascular because of hyperosmolarity 4. Intercellular to intravascular because of increased hydrostatic pressure

3. Intracellular to intravascular because of hyperosmolarity The osmotic effect of hyperglycemia pulls fluid from the intracellular and interstitial compartments, resulting in dehydration. Hyperglycemia pulls fluid from the interstitial to the intravascular compartment, eventually spilling into the urine. Interstitial fluid is part of the extracellular compartment; the osmotic pull of glucose exceeds other osmotic forces. An increase in hydrostatic pressure results in an intravascular to interstitial shift.

An LPN/LVN is assigned to perform assessments on two patients with diabetes. Assessments reveals all of these findings. Which finding would the RN instruct the LPN/LVN to report immediately? 1. Fingerstick glucose reading of 185 mg/dL (10.3 mmol/L) 2. Numbness and tingling in both feet 3. Profuse perspiration 4. Bunion on the left great toe

3. Profuse perspiration Profuse perspiration is a symptom of hypoglycemia, a complication of diabetes that requires urgent treatment. A glucose level of 185 mg/dL (10.3 mmol/L) will need coverage with sliding-scale insulin, but this is not urgent. Numbness and tingling, as well as bunions, are related to the chronic nature of diabetes and are not urgent problems.

A child with type 2 diabetes is scheduled for abdominal surgery. Which factors are most important for the nurse to consider during the postoperative period? Select all that apply. 1. Infection will likely occur at the surgical site. 2. Ketoacidosis frequently occurs later in the postoperative period. 3. The blood glucose level will increase because of the stress of surgery. 4. Urine test results are the most useful gauge of diabetic control after surgery. 5. Diabetic control is usually maintained with insulin after surgery.

3. The blood glucose level will increase because of the stress of surgery. 5. Diabetic control is usually maintained with insulin after surgery. The stress of surgery causes the release of epinephrine and glucocorticoids, which increase the blood glucose level. Most individuals with type 2 diabetes who control their diabetes through diet and exercise require insulin during the recovery period. Although the child with diabetes is at risk for infection, surgical aseptic technique should prevent infection. Ketoacidosis is associated with type 1, not type 2, diabetes. Urine test results are affected by many variables and are not reliable indicators of the blood glucose level.

When obtaining the history of a client recently diagnosed with type 1 diabetes, which symptom would the nurse expect to see? 1. Edema 2. Anorexia 3. Weight loss 4. Hypoglycemic episodes

3. Weight loss Protein and lipid catabolism occur because carbohydrates cannot be used by the cells; this results in weight loss and muscle wasting. Dehydration, not edema, is more likely to occur because of the polyuria associated with hyperglycemia. Polyphagia, not anorexia, occurs with diabetes as the client attempts to meet metabolic needs. Hyperglycemia, not hypoglycemia, is present in both type 1 and type 2 diabetes.

A patient has newly-diagnosed type 2 diabetes. Which task should the RN delegate to an experienced unlicensed assistive personnel (UAP)? 1. Arranging a consult with the dietitian 2. Assessing the patient's insulin injection technique 3. Teaching the patient to use a glucometer to monitor glucose at home 4. Checking the patient's glucose level before each meal

4. Checking the patient's glucose level before each meal The experienced UAP would have been taught to perform tasks such as checking pulse oximetry and glucose checks, and these actions would be part of his or her scope of practice. Arranging for a consult with the dietitian is appropriate for the unit clerk. Teaching and assessing require additional education and should be carried out by licensed nurses.

A client with type 2 diabetes travels frequently and asks how to plan meals during trips. Which is the best response by the nurse? 1. "You can order diabetic foods on most airlines and in restaurants." 2. "Plan your food ahead and carry it with you from home." 3. "Monitor your blood glucose level frequently and eat accordingly." 4. "Choose the foods you normally do and follow your food plan wherever you are."

4. "Choose the foods you normally do and follow your food plan wherever you are." According to an individual's needs, consistency and regularity in the food plan should be maintained; this is a basic principle of dietary management of diabetes. Ordering diabetic foods is not necessary; the client can make selections from regular food choices. Planning food ahead and carrying it from home cannot always be done; it is unnecessary because choices can be made within the food plan. The client should follow the food plan.

A client with type 2 diabetes is admitted to the ambulatory surgery unit for elective cataract surgery. Before surgery the client asks the nurse, "How will my diabetes be managed while I am here?" Which is the best response by the nurse? 1. "What did your surgeon tell you?" 2. "Has the anesthesiologist talked to you yet?" 3. "Your surgeon will write your postoperative prescriptions." 4. "I'm not quite certain I understand what you are asking."

4. "I'm not quite certain I understand what you are asking." The nurse needs to know specifically what the client is asking; this response permits clarification. Asking what the surgeon has said collects more information, but it will not clarify what the client wants to know. Asking what the anesthesiologist has said does not relate to what the client wants to know. The nurse is making an assumption about medical management.

Which statement is accurate when teaching the client with diabetes about foot care? 1. "Remove any corns on your feet." 2. "Wear shoes that are a size larger than your feet." 3. "Examine your feet weekly for potential sores." 4. "Wear synthetic fiber socks when exercising."

4. "Wear synthetic fiber socks when exercising." Research demonstrates that socks with synthetic fibers wick away moisture better than other fabrics when participating in vigorous activities. Self-removal of corns can result in injury to the feet. Shoes that do not fit appropriately will create friction causing sores, blisters, and calluses. The feet should be examined daily, not weekly.

4. Which of the following statements are INCORRECT about Diabetic Ketoacidoisis? A. Extreme Hyperglycemia that presents with blood glucose >600 mg/dL B. Ketones are present in the urine. C. Metabolic acidosis is present with Kussmaul breathing. D. Potassium levels should be at least 3.3 or higher during treatment of DKA with insulin therapy.

4. A

A client with type 1 diabetes receives Humulin R insulin in the morning. Shortly before lunch the nurse identifies that the client is diaphoretic and trembling. Which intervention is appropriate? 1. Administer insulin to the client. 2. Give the client lunch immediately. 3. Encourage the client to drink fluids. 4. Assess the client's blood glucose level.

4. Assess the client's blood glucose level. The client needs glucose, not just fluids. The presence of hypoglycemia should be determined before initiating therapy; Humulin R insulin given in the morning peaks within 4 hours or just before lunchtime. After hypoglycemia is verified, the client should be given an immediate source of glucose. Administering insulin is contraindicated; the client is experiencing adaptations of hypoglycemia, and administering insulin will decrease further an already low blood glucose level. Giving the client lunch may be done after hypoglycemia is determined.

An unlicensed assistive personnel (UAP) tells the nurse that while assisting with the morning care of a postoperative patient with type 2 diabetes who has been given insulin, the patient asked if she will always need to take insulin now. What is the RN's priority for teaching the patient? 1. Explain to the patient that she is now considered to have type 1 diabetes. 2. Tell the patient to monitor fingerstick glucose level every 4 hours after discharge. 3. Teach the patient that a person with type 2 diabetes does not always need insulin. 4. Discuss the relationship between illness and increased glucose levels.

4. Discuss the relationship between illness and increased glucose levels. When a patient with diabetes is ill or has surgery, glucose levels become elevated, and administration of insulin may be necessary. This is a temporary change that usually resolves with recovery from the illness or surgery. Option 3 is correct but does not explain why the patient may currently need insulin. The patient does not have type 1 diabetes, and fingerstick glucose checks are usually prescribed for before meals and at bedtime.

A client with diabetes mellitus who shows decreased glucose tolerance is at risk for which complication? 1. Cystitis 2. Thin and dry skin 3. Decreased bone density 4. Frequent yeast infections

4. Frequent yeast infections Decreased glucose tolerance may cause frequent yeast infections, but it is not associated with the risk of cystitis, thin and dry skin, and decreased bone density. The risk of cystitis, thin and dry skin, and decreased bone density are due to decreased ovarian production of estrogen.

The experienced unlicensed assistive personnel (UAP) has been delegated to take vital signs and check fingerstick glucose on a postoperative patient with diabetes. Which vital sign change would the RN instruct the UAP to report immediately? 1. Blood pressure increase from 132/80 to 138/84 mm Hg 2. Temperature increase from 98.4°F to 99°F (36.9°C to 37.2°C) 3. Respiratory rate increase from 18 to 22 breaths/min 4. Glucose increase from 190 to 236 mg/dL (10.6 to 13.1 mmol/L)

4. Glucose increase from 190 to 236 mg/dL (10.6 to 13.1 mmol/L) An unexpected rise in blood glucose is associated with increased mortality and morbidity after surgical procedures. American Diabetes Association guidelines recommend insulin protocols to maintain blood glucose levels between 140 and 180 mg/dL (7.8 and 10 mmol/L). Also, unexpected rises in blood glucose values may indicate wound infection.

The nurse is reviewing the laboratory report of an adolescent with type 1 diabetes. Which test is considered the most accurate in the evaluation of the effectiveness of diet and insulin therapy over time? 1. Blood pH 2. Serum protein level 3. Serum glucose level 4. Glycosylated hemoglobin

4. Glycosylated hemoglobin The glycosylated hemoglobin (GHb) test provides an accurate long-term index of the child's average blood glucose level for the 10- to 12-day period before the test; the more glucose the red blood cells were exposed to, the greater the GHb percentage. A high blood pH may indicate developing ketoacidosis, but it reflects short-term variations. Serum protein readings do not reflect the effectiveness of glucose management. Serum glucose readings reflect short-term (hours) variations.

A 15-year-old adolescent is found to have type 1 diabetes. Which would the nurse include when teaching the adolescent about type 1 diabetes? 1. It does not always require insulin. 2. It involves early vascular changes. 3. It occurs more often in obese adolescents. 4. It has a more rapid onset than does type 2 diabetes.

4. It has a more rapid onset than does type 2 diabetes. A characteristic difference between type 1 and type 2 diabetes is the rapid onset of type 1 diabetes. Type 1 diabetes often is first diagnosed during an episode of acute ketoacidosis. Children, adolescents, and adults with type 1 diabetes are insulin dependent. Vascular changes are complications associated with long-standing diabetes. Maturity-onset diabetes of the young (MODY), similar to type 2 diabetes, is more often seen in obese teenagers. Adolescents with type 1 diabetes tend to be at or below the expected weight for their height and bone structure.

While working in the diabetes clinic, the RN obtains the following information about an 8-year-old patient with type 1 diabetes. Which finding is most important to address when planning child and parent education? 1. Most recent hemoglobin A1c level of 7.8% 2. Many questions about diet choices from the parents 3. Child's participation in soccer practice after school 2 days a week 4. Morning preprandial glucose range of 55 to 70 mg/dL (3.1 to 3.9 mmol/L)

4. Morning preprandial glucose range of 55 to 70 mg/dL (3.1 to 3.9 mmol/L) The low morning fasting blood glucose level indicates possible nocturnal hypoglycemia. Research indicates that it is important to avoid hypoglycemic episodes in pediatric patients because of the risk for permanent neurologic damage and adverse developmental outcomes. Although a lower hemoglobin A1c might be desirable, the upper limit for hemoglobin A1c levels ranges from 7.5% to 8.5% in pediatric patients. The parents' questions about diet and the child's activity level should also be addressed, but the most urgent consideration is education about the need to avoid hypoglycemia.

Before having surgery, a client with type 1 diabetes insulin requirements are elevated but well controlled. Which insulin requirements would the nurse anticipate for this client postoperatively? 1. Decrease 2. Fluctuate 3. Increase sharply 4. Remain elevated

4. Remain elevated Emotional and physical stress may cause insulin requirements to remain elevated in the postoperative period. Insulin requirements will remain elevated rather than decrease. Fluctuating insulin requirements usually are associated with noncompliance, not surgery. A sharp increase in the client's insulin requirements may indicate sepsis, but this is not expected.

When teaching a client with diabetes about monitoring for episodes of hypoglycemia, which symptom would the nurse include in the teaching plan? 1. Thirst 2. Nausea 3. Anorexia 4. Sweating

4. Sweating When serum glucose decreases, the sympathetic nervous system is stimulated, resulting in a surge of epinephrine and norepinephrine; this response causes sweating, tremors, tachycardia, palpitations, nervousness, and hunger. Increased thirst (polydipsia) occurs in response to the osmotic diuresis associated with hyperglycemia. The ketosis and acidosis of diabetic ketoacidosis lead to gastrointestinal problems such as nausea, anorexia, vomiting, and abdominal cramping.

Which is an appropriate teaching goal for a client who is newly diagnosed as having type 2 diabetes? 1. To perform foot care weekly 2. To administer insulin as prescribed 3. To test urine for both sugar and acetone 4. To identify symptoms of hypoglycemia or hyperglycemia

4. To identify symptoms of hypoglycemia or hyperglycemia Knowledge of the signs and symptoms for hypoglycemia or hyperglycemia is critical to the client's health and well-being and essential for survival. Although performing foot care is important, it should be done daily. The client has type 2 diabetes, which is usually controlled by oral hypoglycemic, not insulin. Self-serum glucose monitoring is more accurate than sugar and acetone urine measurements to identify serum glucose levels.

A patient with type 1 diabetes reports feeling dizzy. What should the nurse do first? 1. Check the patient's blood pressure. 2. Give the patient some orange juice. 3. Give the patient's morning dose of insulin. 4. Use a glucometer to check the patient's glucose level.

4. Use a glucometer to check the patient's glucose level. Before orange juice or insulin is given, the patient's blood glucose level should be checked. Checking blood pressure is a good idea but is not the first action the nurse should take.

5. A patient with Type 2 Diabetes is started on the medication Glyburide. Which of the following statements by the patient causes concern? A. "I will monitor my blood glucose regularly because I know this medication can cause a low blood sugar." B. "I will consume no more than 8 oz. of alcohol per week." C. "I will continue monitoring my diet and participating in exercise while taking this medication." D. "This medication works by stimulating the beta cells in the pancreas to make insulin."

5. B.

6. You are providing care to a patient experiencing diabetic ketoacidosis. The patient is on an insulin drip and their current glucose level is 300. In addition to this, the patient also has 5% Dextroxe 0.45% NS infusing in the right antecubital vein. Which of the following patient signs/symptoms causes concern? A. Patient has a potassium level of 2.3 B. Patient complains of thirst. c. Patient is nauseous. D. Patient's skin and mucous membranes are dry.

6. A

6. A patient is scheduled to take a morning dose of Metformin. The patient is scheduled for surgery tomorrow. Which of the following nursing interventions are correct? A. Administer the medication as ordered. B. Hold the dose and notify the doctor for further orders. C. Administer the medication as ordered but hold tomorrows morning's dose. D. Check the patient's blood glucose prior to administering the medication.

6. B.

The nurse is reviewing laboratory results for the clinic patients to be seen today. Which patient meets the diagnostic criteria for diabetes? A 48-yr-old woman with a hemoglobin A1C of 8.4% A 58-yr-old man with a fasting blood glucose of 111 mg/dL A 68-yr-old woman with a random plasma glucose of 190 mg/dL A 78-yr-old man with a 2-hour glucose tolerance plasma glucose of 184 mg/dL

A 48-yr-old woman with a hemoglobin A1C of 8.4% Criteria for a diagnosis of diabetes include a hemoglobin A1C of 6.5% or greater, fasting plasma glucose level of 126 mg/dL or greater, 2-hour plasma glucose level of 200 mg/dL or greater during an oral glucose tolerance test, or classic symptoms of hyperglycemia or hyperglycemic crisis with a random plasma glucose of 200 mg/dL or greater.

Which patient with type 1 diabetes would be at the highest risk for developing hypoglycemic unawareness? A 58-yr-old patient with diabetic retinopathy A 73-yr-old patient who takes propranolol (Inderal) A 19-yr-old patient who is on the school track team A 24-yr-old patient with a hemoglobin A1C of 8.9%

A 73-yr-old patient who takes propranolol (Inderal) Hypoglycemic unawareness is a condition in which a person does not have the warning signs and symptoms of hypoglycemia until the person becomes incoherent and combative or loses consciousness. Hypoglycemic awareness is related to autonomic neuropathy of diabetes that interferes with the secretion of counterregulatory hormones that produce these symptoms. Older patients and patients who use β-adrenergic blockers (e.g., propranolol) are at risk for hypoglycemic unawareness.

The newly diagnosed patient with type 2 diabetes has been prescribed metformin. What should the nurse teach the patient to explain how this medication works? Increases insulin production from the pancreas. Slows the absorption of carbohydrate in the small intestine. Reduces glucose production by the liver and enhances insulin sensitivity. Increases insulin release from the pancreas and inhibits glucagon secretion.

Reduces glucose production by the liver and enhances insulin sensitivity. Metformin is a biguanide that reduces glucose production by the liver and enhances the tissue's insulin sensitivity. Sulfonylureas and meglitinides increase insulin production from the pancreas. α-Glucosidase inhibitors slow the absorption of carbohydrate in the intestine. Glucagon-like peptide receptor agonists increase insulin synthesis and release from the pancreas, inhibit glucagon secretion, and decrease gastric emptying.

4. Which of the following patients is at most risk for Type 2 diabetes?* A. A 6 year old girl recovering from a viral infection with a family history of diabetes. B. A 28 year old male with a BMI of 49. C. A 76 year old female with a history of cardiac disease. D. None of the options provided

The answer is B. Remember Type 2 diabetes risk factors are related to lifestyle....being obese is a risk factor (BMI >30 in males is considered obese). So, the 28 year old male with a BMI of 49 is most at risk for Type 2.


Conjuntos de estudio relacionados

Prep U Chapter 34: Assessment and Management of Patients with Inflammatory Rheumatic Disorders

View Set

Flashcard for Chapter 6 Terms and Definitions

View Set

3/4 adrenal gland hormones and regulations

View Set

Chapter 3: Interdependence and the Gains from Trade

View Set